[obm-l] Re: [obm-l] Re: [obm-l] Re: [obm-l] Re: [obm-l] Re: [obm-l] Questão de probabilidade

2022-06-24 Por tôpico Rogerio Ponce
Otima explicacao! Obrigado, Ralph! PS: e sim, a provocacao foi pra voce mesmo! :) []'s Rogerio Ponce On Wed, Jun 22, 2022 at 1:00 PM Ralph Costa Teixeira wrote: > > Ponce está provocando a gente... senti que esta flecha tinha um bocado a > minha direção... :D :D :D > > Olha, tem duas "visões

[obm-l] Re: [obm-l] Re: [obm-l] Re: [obm-l] Re: [obm-l] Questão de probabilidade

2022-06-22 Por tôpico Ralph Costa Teixeira
Ponce está provocando a gente... senti que esta flecha tinha um bocado a minha direção... :D :D :D Olha, tem duas "visões" sobre o que "probabilidade" significa. A primeira vai na linha de que só podemos falar de probabilidade sobre coisas que ainda não aconteceram. Vai nessa linha: se os evento

[obm-l] Re: [obm-l] Re: [obm-l] Re: [obm-l] Questão de probabilidade

2022-06-22 Por tôpico Rogerio Ponce
Olá Pedro e pessoal da lista! Segundo a opinião do Pedro, nao faz sentido perguntar qual a probabilidade de Jose ter conseguido um 6 ao jogar o dado ontem, pois isso ja' aconteceu, e, portanto, ja' esta' definido. Sera' que e' isso mesmo? []'s Rogerio Ponce On Mon, Jun 20, 2022 at 9:45 PM Pedr

[obm-l] Re: [obm-l] Re: [obm-l] Questão de probabilidade

2022-06-20 Por tôpico Pedro José
Eu na minha humilde opinião creio que a probabilidade exista quando pode ser uma coisa ou outra. No caso já é definido o que os animais são. Então já está tudo errado. A questão seria viável se dessem esses limitantes para uma criança que pintaria os desenhos dos animais. Aí sim há probabilidade.

[obm-l] Re: [obm-l] Re: [obm-l] Questão sobre desigualdades

2021-04-14 Por tôpico Carlos Monteiro
De onde saiu essa desigualdade? Em qua., 14 de abr. de 2021 às 20:39, Anderson Torres < torres.anderson...@gmail.com> escreveu: > Em qua., 14 de abr. de 2021 às 15:54, Carlos Monteiro > escreveu: > > > > Encontre os valores máximo e mínimo da expressão: x/(x^2+1) + y/(y^2+1) > + z/(z^2+1) , ond

[obm-l] Re: [obm-l] Re: [obm-l] Re: [obm-l] Questão OBM - U

2020-01-23 Por tôpico Ralph Teixeira
Seja ABCD o quadrilatero convexo, e seja P o encontro das diagonais. No triangulo APB, temos AP+PB>AB. Escreva as desigualdades analogas para os triangulos BPC, CPD e DPA. Somando-as, voce vai obter que 2(AC+BD)>perimetro=8 Ou seja, o infimo tem que ser pelo menos 4. Agora, para chegar no infim

[obm-l] Re: [obm-l] Re: [obm-l] Re: [obm-l] Re: [obm-l] Re: [obm-l] Re: [obm-l] Questão OBM - U

2020-01-23 Por tôpico Esdras Muniz
É fácil ver que esse ínfimo tem que ser no mínimo 4, basta fazer desigualdade triângulos com os triângulos que têm dois vértices comuns com o quadrilátero e o terceiro sendo a interseção das diagonais. E por esse argumento do Caio, vemos que é 4 mesmo. Em qui, 23 de jan de 2020 08:59, Caio Costa

[obm-l] Re: [obm-l] Re: [obm-l] Re: [obm-l] Re: [obm-l] Re: [obm-l] Questão OBM - U

2020-01-23 Por tôpico Caio Costa
Minimiza-se a soma das diagonais ao tomar-se um losango degenerado, com uma diagonal valendo 4 e outra valendo 0. Em qui, 23 de jan de 2020 08:34, gilberto azevedo escreveu: > Pensei em minimizar √(a² + (4-a)²) > 4 - a, devido ao fato do perímetro ser 8. > No caso obtenho o mínimo sendo 2√2, qua

[obm-l] Re: [obm-l] Re: [obm-l] Re: [obm-l] Re: [obm-l] Questão OBM - U

2020-01-23 Por tôpico gilberto azevedo
Pensei em minimizar √(a² + (4-a)²) 4 - a, devido ao fato do perímetro ser 8. No caso obtenho o mínimo sendo 2√2, quando o retângulo é um quadrado de lado 2. A soma das diagonais seria no caso 4√2, e não bate com o gabarito. Em qui, 23 de jan de 2020 08:20, Bernardo Freitas Paulo da Costa < bernard

[obm-l] Re: [obm-l] Re: [obm-l] Re: [obm-l] Questão OBM - U

2020-01-23 Por tôpico Bernardo Freitas Paulo da Costa
On Thu, Jan 23, 2020 at 7:24 AM gilberto azevedo wrote: >> On Sat, Jan 11, 2020 at 11:24 AM gilberto azevedo >> wrote: >> > >> > Qual o ínfimo sobre todos os quadriláteros convexos com perímetro 8 da >> > soma dos comprimentos de suas diagonais ? > > Tentei com o retângulo e o quadrado, poré

[obm-l] Re: [obm-l] Re: [obm-l] Questão OBM - U

2020-01-23 Por tôpico gilberto azevedo
Tentei com o retângulo e o quadrado, porém não obtive a resposta... O gabarito é 4. Em sáb, 11 de jan de 2020 12:03, Bernardo Freitas Paulo da Costa < bernardo...@gmail.com> escreveu: > On Sat, Jan 11, 2020 at 11:24 AM gilberto azevedo > wrote: > > > > Qual o ínfimo sobre todos os quadriláter

[obm-l] Re: [obm-l] Re: [obm-l] Questão de probabilidade

2018-11-06 Por tôpico Vanderlei Nemitz
Bela solução, Bruno! Muito obrigado! Em ter, 6 de nov de 2018 15:38, Bruno Visnadi Seja Pa a probabilidade de ocorrência de a. Defina Pb e Pc analogamente. > a = Pa(1-Pb)(1-Pc) > b = Pb(1-Pa)(1-Pc) > c = Pc(1-Pa)(1-Pb) > p = (1-Pa)(1-Pb)(1-Pc) > Queremos achar a razão Pa/Pc > Da equação (a - 2b)p

[obm-l] Re: [obm-l] Re: [obm-l] Questão do ITA

2018-10-11 Por tôpico Vanderlei Nemitz
Valeu, Ralph! Como sempre, uma explicação clara e simples! Em qua, 10 de out de 2018 17:05, Ralph Teixeira escreveu: > Note que x=5 é um possível valor que resolve aquela equação (mas, > sinceramente, não interessa, eu faria o raciocínio abaixo com qualquer > número). > > Então qualquer polinômi

[obm-l] Re: [obm-l] Re: [obm-l] Questão do IME

2018-07-14 Por tôpico Vanderlei Nemitz
Muito obrigado, Claudio! Bela solução! Em 13 de julho de 2018 13:35, Claudio Buffara escreveu: > Os prolongamentos de DM e EN se intersectam num mesmo ponto P pertencente > a AB. > Pra ver isso, repare que os triângulos DCM e PAM são semelhantes (razão de > semelhança = 2). > Idem para os triâng

[obm-l] Re: [obm-l] Re: [obm-l] Questão sobre divisor primo

2018-06-06 Por tôpico Claudio Buffara
Na verdade é pra provar que se p é primo e divide 12n^2 + 1, então p é de forma 6k+1. 2018-06-06 12:50 GMT-03:00 Daniel Quevedo : > De uma maneira bem informal 6| 12n^2 , para qqr n inteiro. Logo 12n^2+1= 1 > (mod 6) ou seja é da forma 6k +1. > > Uma demonstração formal seria por indução finita

[obm-l] Re: [obm-l] Re: [obm-l] Re: [obm-l] Re: [obm-l] Questão de derivada

2018-04-23 Por tôpico Igor Caetano Diniz
na verdade eu não fiz rsrs. Eu queria ver um modo claro de mostrar. Se não puder usar L'Hospital, acho que tem que fazer uma sequência por baixo e uma por cima aplicando TVM em cada intervalo. Aí usa o fato dessa sequencia ser limitada, e monotona, portanto, convergente. Logo lim f'(xn) = L tanto

[obm-l] Re: [obm-l] Re: [obm-l] Re: [obm-l] Questão de derivada

2018-04-23 Por tôpico Artur Steiner
Eu li errado, temos que lim x --> 0 f' (x) = L. Assim, a Regra de l' Hopital conforme mostrei demonstra que, de fato, f'(c) = L. Mas o que vc fez não mostra que f'(c) = L. Artur Costa Steiner Em Seg, 23 de abr de 2018 14:31, Igor Caetano Diniz escreveu: > Se a questão tivesse um intervalo exp

[obm-l] Re: [obm-l] Re: [obm-l] Questão de derivada

2018-04-23 Por tôpico Igor Caetano Diniz
Se a questão tivesse um intervalo explícito [a,b] e diferenciável em todo ponto (a,b) exceto possivelmente num ponto c em (a,b) tal que lim f '(x) = L, x-> c, o que eu fiz estaria correto? 2018-04-23 14:11 GMT-03:00 Artur Steiner : > Como f é contínua em 0, então, pela regra de L'Hopital, > > lim

[obm-l] Re: [obm-l] Re: [obm-l] Questão de derivada

2018-04-23 Por tôpico Igor Caetano Diniz
Então, Se existem os limites laterais, lim f ' (0-) = lim f ' (0+) então, defina q(x) = [f(x) - f(0)]/x. Para todo x<0, existe y1 entre x e 0 tal que f ' (y) = q(x). Analogamente para x>0, existe z1 entre 0 e x tal que f ' (z) = q(x). Defina r(x,0) a distancia de x para 0 Então, seja yn = yn-1 + r

[obm-l] Re: [obm-l] Re: [obm-l] Re: [obm-l] Re: [obm-l] Questão de Combinatória

2018-03-30 Por tôpico Anderson Torres
Em 29 de março de 2018 15:37, Igor Caetano Diniz escreveu: > Vou mostrar a sua e a minha e aí se ele não aprender com as duas, tento > fazer devagar em casos menores. hehe > > Abraços Cláudio e obrigado =) > > 2018-03-29 15:17 GMT-03:00 Claudio Buffara : >> >> Sim. Acho essa uma solução bem mais e

[obm-l] Re: [obm-l] Re: [obm-l] Re: [obm-l] Re: [obm-l] Questão de Combinatória

2018-03-29 Por tôpico Claudio Buffara
Outra sugestão: proponha o problema de contar de quantas maneiras é possível arrumar N dominós 1x2 numa caixa 2xN. Fibonacci também aparece neste aí. A diferença é que, no dos bits, B(N) = F(N+2) enquanto que, no dos dominós, D(N) = F(N+1) (F é definida da forma usual, com F(1) = F(2) = 1) Ou ent

[obm-l] Re: [obm-l] Re: [obm-l] Questão de Combinatória

2018-03-29 Por tôpico Claudio Buffara
Sugestão de natureza didática: eu mostraria uma solução mais braçal, tal como a minha, e depois mostraria a solução recursiva. Moral: em geral vale a pena pensar no problema antes de sair escrevendo... 2018-03-29 15:17 GMT-03:00 Claudio Buffara : > Sim. Acho essa uma solução bem mais elegante. >

[obm-l] Re: [obm-l] Re: [obm-l] Re: [obm-l] Questão de Combinatória

2018-03-29 Por tôpico Igor Caetano Diniz
Vou mostrar a sua e a minha e aí se ele não aprender com as duas, tento fazer devagar em casos menores. hehe Abraços Cláudio e obrigado =) 2018-03-29 15:17 GMT-03:00 Claudio Buffara : > Sim. Acho essa uma solução bem mais elegante. > Mas também é mais sofisticada, e você falou que o aluno é prin

[obm-l] Re: [obm-l] Re: [obm-l] Questão de Combinatória

2018-03-29 Por tôpico Claudio Buffara
Sim. Acho essa uma solução bem mais elegante. Mas também é mais sofisticada, e você falou que o aluno é principiante. De todo jeito, acho que raciocinar recursivamente é uma habilidade que todo estudante de matemática deveria desenvolver. []s, Claudio. 2018-03-29 14:45 GMT-03:00 Igor Caetano Di

[obm-l] Re: [obm-l] Re: [obm-l] Re: [obm-l] Re: [obm-l] Re: [obm-l] Questão de Cardinalidade

2018-01-21 Por tôpico Anderson Torres
Em 16 de janeiro de 2018 13:50, Bernardo Freitas Paulo da Costa escreveu: > 2018-01-16 1:10 GMT-02:00 Anderson Torres : >> Eu na verdade pensei ao contrário: >> >> Começamos com o conjunto de todos os subconjuntos de N. Cada conjunto >> será representado por uma string infinita de zeros e unzes, d

[obm-l] Re: [obm-l] Re: [obm-l] Re: [obm-l] Re: [obm-l] Re: [obm-l] Re: [obm-l] Questão de Cardinalidade

2018-01-16 Por tôpico Bernardo Freitas Paulo da Costa
2018-01-16 14:11 GMT-02:00 Igor Caetano Diniz : > Fala Bernardo, tudo certo? > Mas sera que eu conseguiria provar que esses números não seriam uma > quantidade enumeravel de pontos entre 0 e 1 e, então, como é enumeravel, eu > consigo pegar uma quantidade enumeravel em P(N) para esses pontos. Sim,

[obm-l] Re: [obm-l] Re: [obm-l] Re: [obm-l] Re: [obm-l] Re: [obm-l] Questão de Cardinalidade

2018-01-16 Por tôpico Igor Caetano Diniz
Fala Bernardo, tudo certo? Mas sera que eu conseguiria provar que esses números não seriam uma quantidade enumeravel de pontos entre 0 e 1 e, então, como é enumeravel, eu consigo pegar uma quantidade enumeravel em P(N) para esses pontos. Acha que seria ruim? Abraço On Jan 16, 2018 13:59, "Bernard

[obm-l] Re: [obm-l] Re: [obm-l] Re: [obm-l] Re: [obm-l] Questão de Cardinalidade

2018-01-16 Por tôpico Bernardo Freitas Paulo da Costa
2018-01-16 1:10 GMT-02:00 Anderson Torres : > Eu na verdade pensei ao contrário: > > Começamos com o conjunto de todos os subconjuntos de N. Cada conjunto > será representado por uma string infinita de zeros e unzes, da > seguinte forma: Se o conjunto contiver o natural x, o x-ésimo > caractere des

[obm-l] Re: [obm-l] Re: [obm-l] Re: [obm-l] Re: [obm-l] Questão de Cardinalidade

2018-01-16 Por tôpico Igor Caetano Diniz
Uma ideia legal Para provar que (-1,1) tem bijeção com R, seria usar f(x) = x/(x^2-1) provando que ela eh injetiva e sobrejetiva On Jan 16, 2018 01:20, "Anderson Torres" wrote: > Eu na verdade pensei ao contrário: > > Começamos com o conjunto de todos os subconjuntos de N. Cada conjunto > será r

[obm-l] Re: [obm-l] Re: [obm-l] Re: [obm-l] Questão de Cardinalidade

2018-01-15 Por tôpico Anderson Torres
Eu na verdade pensei ao contrário: Começamos com o conjunto de todos os subconjuntos de N. Cada conjunto será representado por uma string infinita de zeros e unzes, da seguinte forma: Se o conjunto contiver o natural x, o x-ésimo caractere desta string será 1; caso contrário, será 0. Botando zero

[obm-l] Re: [obm-l] Re: [obm-l] Questão de Cardinalidade

2018-01-15 Por tôpico Igor Caetano Diniz
Olá Sávio, Muito obrigado. Tava pensando em algo parecido mas agora voce esclareceu bastante. Abraços On Jan 15, 2018 16:55, "Sávio Ribas" wrote: > Boa tarde! > A primeira parte servirá para mostrar que a cardinalidade de IR é igual à > cardinalidade de [0,1]. > Não é difícil mostrar que a reta

[obm-l] Re: [obm-l] Re: [obm-l] Questão

2017-08-28 Por tôpico Pedro José
Bom dia! Daniel, eu já me sinto gratificado quando consigo resolver algo. Não sou matemático, sou um pitaqueiro, com alto grau curiosidade e matemática é uma das minhas curiosidades preferidas. O que mais me fascina, é que sou totalmente crente em que um modelo matemático formulado com estrutura,

[obm-l] Re: [obm-l]Re: [obm-l] Re: [obm-l] Re: [obm-l] Questão de teoria numérica

2017-08-01 Por tôpico Pedro Cardoso
Realmente. Se isso serve de desculpa eu escrevi isso assim que acordei. O que eu quis dizer é que não existem múltiplos de 2017 que terminem em 0 e que, ao serem divididos por 10, deixam de ser múltiplos de 2017. Para isso existir, 2017 teria que ter um número de fatores 2 diferente do número de

[obm-l] Re: [obm-l] Re: [obm-l] Re: [obm-l] Questão de teoria numérica

2017-08-01 Por tôpico Bernardo Freitas Paulo da Costa
Em 01/08/2017 08:14, "Pedro Cardoso" escreveu: > > Obrigado! Era exatamente isso que a questão anterior sugeria, usar o > princípio da casa dos pombos. > Uma coisa que percebi na sua dsmonstração é que o número encontrado > terminaria em 0s, mas como nenhum multiplo de 2017 também é multiplo de

[obm-l] Re: [obm-l] Re: [obm-l] Questão de teoria numérica

2017-08-01 Por tôpico Pedro Cardoso
Obrigado! Era exatamente isso que a questão anterior sugeria, usar o princípio da casa dos pombos. Uma coisa que percebi na sua dsmonstração é que o número encontrado terminaria em 0s, mas como nenhum multiplo de 2017 também é multiplo de 10 (2017 é primo) então também existe um multiplo de 2017 co

[obm-l] Re: [obm-l] Re: [obm-l] Questão de Física

2016-10-16 Por tôpico Luiz Antonio Rodrigues
Olá, Pacini! Muito obrigado! Um abraço! Luiz On Oct 16, 2016 10:38 AM, "Pacini Bores" wrote: > > > > Oi Luiz, > > o T para pequenas oscilações , T = 2.pi.sqrt(L/g) e com T´=5T= > 2.pi.sqrt(L/g´), onde g´= (P-q.E)/m. > > Logo teremos : (T^2).g = ((T´)^2).g´ ou seja g=25.g´ou g = 25(P-q.E)/m e > fa

[obm-l] Re: [obm-l] Re: [obm-l] Questão de vetores

2016-09-21 Por tôpico Bruno Lira
Tome N um ponto tal que MN seja paralelo a AB. Note que o triângulo ABC é semelhante ao triângulo NMC. Dá semelhança de triângulo temos que: NM/AB = MC/BC => NM/BC = 5/8 => NM =5AB/8. e NC/AC = MC/BC => NC/AC = 5/8 => NC = 5AC/8. AN = AC -NC = 3AC/8. Daí: vetor(AM) = vetor(AN) + vetor(NM)

[obm-l] Re: [obm-l] Re: [obm-l] questão de treinamento olimpica.

2015-10-07 Por tôpico Israel Meireles Chrisostomo
Ah tah, agora que eu vi que é o produto Em 7 de outubro de 2015 13:57, Israel Meireles Chrisostomo < israelmchrisost...@gmail.com> escreveu: > Mas isso aí não pode ser resolvido pelo princípio da casa dos pombos? > > > Em 7 de outubro de 2015 10:29, Esdras Muniz > escreveu: > >> Supondo por absu

[obm-l] Re: [obm-l] Re: [obm-l] questão de treinamento olimpica.

2015-10-07 Por tôpico Israel Meireles Chrisostomo
Mas isso aí não pode ser resolvido pelo princípio da casa dos pombos? Em 7 de outubro de 2015 10:29, Esdras Muniz escreveu: > Supondo por absurdo que isso ocorra, daí temos que se a_i=11, então > b_i=11, do contrario, teríamos dois produtos de resto zero por 11. Então > vamos supor sem perda d

[obm-l] Re: [obm-l] Re: [obm-l] Re: [obm-l] Questão Colégio Militar - anulada

2015-07-27 Por tôpico Mauricio de Araujo
Na verdade eu concordo com as soluções só estou atrás de um argumento que justifique a anulação da questão... a única possibilidade que encontrei foi alguém desenvolver um raciocínio diferente em cima da inclusão ou não dos 5 minutos nos 10 minutos de atraso... []'s Em 27 de julho de 2015 14:1

[obm-l] Re: [obm-l] Re: [obm-l] Questão Colégio Militar - anulada

2015-07-27 Por tôpico Pedro José
Boa tarde! Desculpe-me, mas a Kacilda, faz o julgamento com as suas premissas. Obviamente, se ela pensa eu o relógio está atrasado ela incluirá os 5 min. Sds, PJMS Em 27 de julho de 2015 13:43, Mauricio de Araujo < mauricio.de.ara...@gmail.com> escreveu: > > Em 25 de julho de 2015 22:17, Marti

[obm-l] Re: [obm-l] Re: [obm-l] Questão Colégio Militar - anulada

2015-07-26 Por tôpico Ralph Teixeira
P.S.: Oops, erro tipografico: troque aquele "10 minutos" do finalzinho por 15. O resto estah ok: sao, de fato, 11h50m. 2015-07-26 14:18 GMT-03:00 Ralph Teixeira : > Para mim, letra (a). > > O problema eh esquisito porque a lingua portuguesa parece esquisita > Afinal: > > Suponha que voce tem

[obm-l] Re: [obm-l] Re: [obm-l] Questão Colégio Militar - anulada

2015-07-26 Por tôpico Ralph Teixeira
Para mim, letra (a). O problema eh esquisito porque a lingua portuguesa parece esquisita Afinal: Suponha que voce tem um encontro na hora x. Se voce estah ATRASADO y minutos, voce chegou na hora x+y. Se voce estah ADIANTADO y minutos, voce chegou na hora x-y. A principio, a linguagem PARECE

[obm-l] Re: [obm-l] Re: [obm-l] Questão Colégio Militar - anulada

2015-07-26 Por tôpico Pedro José
Bom dia! Também não vejo uma razão para a anulação. Porém se o relógio está 15 min adiantado e Kacilda acha que está 5min atrasado. A referência Kacilda está 20min adiantada. Se ela julga estar 10 min atrasada na verdade ela está 10 min adiantada. Letra "a)". Sds, PJMS Em 26 de julho de 2015 12:

[obm-l] Re: [obm-l] Re: [obm-l] Questão de probabilidade

2015-01-18 Por tôpico Ralph Teixeira
Eh, o Hermann tem razao, nao existe uma distribuicao de probabilidade nos reais positivos que "funcione" bem. O problema eh que existem varias maneiras de "escolher um numero real positivo aleatoriamente", nenhuma delas completamente padrao, e elas dariam respostas diferentes para seu problema. ---

[obm-l] Re: [obm-l] Re: [obm-l] Questão da 3ª fase nível 1 da OBM 2013

2014-04-26 Por tôpico saulo nilson
222 4 1 6 12 9 18 3 b) 8 412 2 1 3 105 15 2014-04-23 23:13 GMT-03:00 Willy George Amaral Petrenko < wgapetre...@gmail.com>: > vc quer uma ajuda ou uma solução? > > Uma ajuda: > > a) Observe que 2

[obm-l] Re: [obm-l] Re: [obm-l] Re: [obm-l] Re: [obm-l] Re: [obm-l] Questão do IME

2013-10-29 Por tôpico Leonardo Borges Avelino
O IME sempre teve costume de usar questões de livros famosos, como Lidski, Caronnet e outros. Realmente é muito difícil ter acesso a estes livros (em papel), pois são caros. Muitos tem como achar na internet em PDF, mas eu sempre gostei de ter os livros. Recomendo a seguinte engine de busca de liv

[obm-l] Re: [obm-l] Re: [obm-l] Re: [obm-l] Re: [obm-l] Questão do IME

2013-10-29 Por tôpico Mauricio de Araujo
utor é Suprun tem hoje a venda por um absurdo de preço na > editora Vestseller > > - Original Message - > *From:* Vanderlei Nemitz > *To:* obm-l@mat.puc-rio.br > *Sent:* Tuesday, October 29, 2013 4:13 PM > *Subject:* [obm-l] Re: [obm-l] Re: [obm-l] Questão do IME >

[obm-l] Re: [obm-l] Re: [obm-l] Re: [obm-l] Questão do IME

2013-10-29 Por tôpico Hermann
O autor é Suprun tem hoje a venda por um absurdo de preço na editora Vestseller - Original Message - From: Vanderlei Nemitz To: obm-l@mat.puc-rio.br Sent: Tuesday, October 29, 2013 4:13 PM Subject: [obm-l] Re: [obm-l] Re: [obm-l] Questão do IME Maurício: Que livro é

[obm-l] Re: [obm-l] Re: [obm-l] Questão do IME

2013-10-29 Por tôpico Vanderlei Nemitz
Maurício: Que livro é esse? O IME retirou a questão na íntegra. Obrigado! Em 29 de outubro de 2013 15:56, Mauricio de Araujo < mauricio.de.ara...@gmail.com> escreveu: > veja a solução em > > https://www.dropbox.com/s/3wpkb4ht01oidsz/foto%205.PNG > > > 2013/10/29 marcone augusto araújo borges

[obm-l] Re: [obm-l] RE: [obm-l] Re: [obm-l] Re: [obm-l] questão bacana(quase me tira o sono)

2013-06-13 Por tôpico Lucas Prado Melo
2013/6/13 Lucas Prado Melo > > Observando o somatório, temos que F(n) está sendo somado por vários termos > na forma g(n) F(i)/(i! 2^i) onde f é uma função. > Quando observamos o mesmo para F(n+1) os termos com fatores F(i)/(i! 2^i) > ainda aparecem, mas o coeficiente muda: g(n+1) = 2n g(n). > >

[obm-l] Re: [obm-l] Re: [obm-l] Re: [obm-l] Re: [obm-l] Re: [obm-l] questão bacana(quase me tira o sono)

2013-06-13 Por tôpico Cassio Anderson Feitosa
Ah sim. Acabei interpretando o questão de forma errada também. Pensei que depois de colocar todos os pesos é que ia ser verificado o peso dos pratos. Em 13 de junho de 2013 12:42, Lucas Prado Melo escreveu: > > 2013/6/13 Cassio Anderson Feitosa > >> Eu pensei também no problema e vou mostrar o

[obm-l] Re: [obm-l] RE: [obm-l] Re: [obm-l] Re: [obm-l] questão bacana(quase me tira o sono)

2013-06-13 Por tôpico Lucas Prado Melo
2013/6/13 Lucas Prado Melo > > Observando o somatório, temos que F(n) está sendo somado por vários termos > na forma g(n) F(i)/(i! 2^i) onde f é uma função. > Quando observamos o mesmo para F(n+1) os termos com fatores F(i)/(i! 2^i) > ainda aparecem, mas o coeficiente muda: g(n+1) = 2n g(n). > >

[obm-l] Re: [obm-l] RE: [obm-l] Re: [obm-l] Re: [obm-l] questão bacana(quase me tira o sono)

2013-06-13 Por tôpico Lucas Prado Melo
2013/6/13 marcone augusto araújo borges > Olá,Lucas > Não entendi bem a passagem ´´...a colocação das i-1 bolinhas menores não > afetariam em nada o cálculo...´´ > Ok eu viajei um pouco nesse trecho. Eu quis dizer que as i-1 bolinhas poderiam ser colocadas livremente. Não importa mais se elas vã

[obm-l] RE: [obm-l] Re: [obm-l] Re: [obm-l] questão bacana(quase me tira o sono)

2013-06-13 Por tôpico marcone augusto araújo borges
dasbolinhas maiores: F(n-i)Obrigado. From: luca...@dcc.ufba.br Date: Tue, 11 Jun 2013 16:06:16 -0300 Subject: [obm-l] Re: [obm-l] Re: [obm-l] questão bacana(quase me tira o sono) To: obm-l@mat.puc-rio.br 2013/6/11 Henrique Rennó Acho que a solução que coloquei está errada. Pensando nos

[obm-l] Re: [obm-l] Re: [obm-l] Re: [obm-l] Re: [obm-l] questão bacana(quase me tira o sono)

2013-06-13 Por tôpico Lucas Prado Melo
2013/6/13 Cassio Anderson Feitosa > Eu pensei também no problema e vou mostrar o que pensei pra que possam me > mostrar o erro, se houver. > > Como 2^0+2^1 + . . . + 2^{99} = 2^{100} -1 < 2^{100}, então não > importa a forma que distribuímos os pesos, o prato com 2^{100} gramas > sempre será

[obm-l] Re: [obm-l] Re: [obm-l] Re: [obm-l] questão bacana(quase me tira o sono)

2013-06-13 Por tôpico Cassio Anderson Feitosa
Eu pensei também no problema e vou mostrar o que pensei pra que possam me mostrar o erro, se houver. Como 2^0+2^1 + . . . + 2^{99} = 2^{100} -1 < 2^{100}, então não importa a forma que distribuímos os pesos, o prato com 2^{100} gramas sempre será mais pesado. Então, o peso com 2^{100} gramas d

[obm-l] Re: [obm-l] Re: [obm-l] questão bacana(quase me tira o sono)

2013-06-11 Por tôpico Lucas Prado Melo
2013/6/11 Henrique Rennó > Acho que a solução que coloquei está errada. Pensando nos expoentes de > forma crescente: se for apenas o peso 2^0 ele tem que estar no prato da > direita. Acrescentando o peso 2^1, ele deve ir para o prato da direita e o > peso anterior tem 2^1 possibilidades. Acrescen

[obm-l] Re: [obm-l] Re: [obm-l] Re: [obm-l] Re: [obm-l] Re: [obm-l] questão sobre invariantes - alguém poderia ajudar?

2013-02-25 Por tôpico Lucas Prado Melo
2013/2/25 Mauricio de Araujo > opa, tua solução também é muito boa sem dúvida... obrigado pelo retorno, > estava sem nenhuma ideia... citei a do Ralph apenas por uma questão de > afinidade com o pensamento apresentado, só isso... > > De boa. :) Tinha imaginado. -- []'s Lucas

[obm-l] Re: [obm-l] Re: [obm-l] Re: [obm-l] questão sobre invariantes - alguém poderia ajudar?

2013-02-24 Por tôpico Lucas Prado Melo
2013/2/24 Lucas Prado Melo > 2013/2/24 Mauricio de Araujo > >> Obrigado a todos pelas orientações... acredito que a ideia do Ralph está >> mais adequada por usar invariância que é o recurso solicitado na resolução. >> >> A minha solução não? > A propósito, só pra esclarecer: eu achei a solução

[obm-l] Re: [obm-l] Re: [obm-l] Re: [obm-l] questão sobre invariantes - alguém poderia ajudar?

2013-02-24 Por tôpico Lucas Prado Melo
2013/2/24 Mauricio de Araujo > Obrigado a todos pelas orientações... acredito que a ideia do Ralph está > mais adequada por usar invariância que é o recurso solicitado na resolução. > > A minha solução não? -- []'s Lucas

[obm-l] Re: [obm-l] RE: [obm-l] Re: [obm-l] Questão sobre número primo

2012-11-05 Por tôpico terence thirteen
Em 4 de novembro de 2012 20:14, marcone augusto araújo borges escreveu: > Me parece ai uma idéia parecida com aquela usada para provar que existem > infinitos primos. > Desconfio que o número é primo e o seu único divisor primo é ele mesmo. > Agente percebe que esse número é maior do que n > e men

[obm-l] RE: [obm-l] Re: [obm-l] Questão sobre número primo

2012-11-04 Por tôpico marcone augusto araújo borges
Me parece ai uma idéia parecida com aquela usada para provar que existem infinitos primos.Desconfio que o número é primo e o seu único divisor primo é ele mesmo.Agente percebe que esse número é maior do que ne menor do que n!,mas acho que eu não saberia formalizar isso.Seja m o produto dos prim

[obm-l] Re: [obm-l] Re: [obm-l] Questão sobre número primo

2012-11-02 Por tôpico terence thirteen
Em 1 de novembro de 2012 22:27, Ralph Teixeira escreveu: > Dica: tome todos os primos menores que n, multiplique e some 1. Quem sao os > divisores prmos deste numero? > > Abraco, > Ralph > > > 2012/11/1 marcone augusto araújo borges >> >> Seja n>2.Mostre que entre n e n! existe pelo menos um

[obm-l] RE: [obm-l] RE: [obm-l] Re: [obm-l] RE: [obm-l] Re: [obm-l] Questão quaternios difícil

2012-09-03 Por tôpico Samuel Wainer
] Re: [obm-l] RE: [obm-l] Re: [obm-l] Questão quaternios difícil Date: Tue, 28 Aug 2012 18:10:36 + Consegui mostrar que M_2(Z/p) é simples. Supus que um ideal continha um elemento não nulo. Supus por exemplo que a primeira entrada da matriz é não nula. Multipliquei por algumas matrizes

[obm-l] RE: [obm-l] Re: [obm-l] RE: [obm-l] Re: [obm-l] Questão quaternios difícil

2012-08-28 Por tôpico Samuel Wainer
: Mon, 27 Aug 2012 21:53:11 -0300 Subject: [obm-l] Re: [obm-l] RE: [obm-l] Re: [obm-l] Questão quaternios difícil From: hit0...@gmail.com To: obm-l@mat.puc-rio.br Vou te revelar a cruel verdade, prepare-se. O que acontece é que, na realidade, o anel de quaternios sobre Z/p vai ser isomorfo, como

[obm-l] Re: [obm-l] RE: [obm-l] Re: [obm-l] Questão quaternios difícil

2012-08-27 Por tôpico Tiago
Vou te revelar a cruel verdade, prepare-se. O que acontece é que, na realidade, o anel de quaternios sobre Z/p vai ser isomorfo, como anel, ao anel M_2(Z/p) (matrizes quadradas sobre Z_p). *Deve* existir uma maneira ad-hoc de encontrar esse isomorfismo. Você pode tentar (eu nunca tentei, mas se voc

[obm-l] RE: [obm-l] Re: [obm-l] Questão quaternios difícil

2012-08-27 Por tôpico Samuel Wainer
Olá com as dicas consegui fazer uma boa parte. Para mostrar que não é um anel de divisão considerei Z = a0 + a1 i + a2 j + a3 k. E considerei z = a0 - a1 i - a2 j - a3 k. Assim Z z = (a0)² + (a1)² + (a2)² + (a3)². Assim um elemento Z vai ter inverso multiplicativo se e somente se (a0)² + (a1

[obm-l] RE: [obm-l] Re: [obm-l] Questão de combinatoria

2012-06-06 Por tôpico marcone augusto araújo borges
Beleza!Eh isso mesmo. Eu tinha pensado em escolher duas linhas e duas colunas,para formar um retângulo. Date: Wed, 6 Jun 2012 00:58:24 -0300 Subject: [obm-l] Re: [obm-l] Questão de combinatoria From: ralp...@gmail.com To: obm-l@mat.puc-rio.br Escolher um retangulo significa escolher o cant

[obm-l] Re: [obm-l] Re: [obm-l] questão obm2010

2012-02-24 Por tôpico Rafael Cano
Deve ser nível <= 3 porque é teste :) Mas os números são tão "desiguais" que dá pra usar uma aproximação beeem mais simples com os logs. Acho que só precisaria saber que log é função injetiva crescente (ou seja, pode "mandar" log dos dois lados da desigualdade :P) Começando com n^log n < (log n)^n

[obm-l] RE: [obm-l] RE: [obm-l] Questão simples

2011-12-26 Por tôpico Eduardo Wilner
O menor n é mesmo 8. n = 13 não satisfaz. Pode ser verificado, por exemplo, seguindo o raciocínio do Bernardo Freitas Paulo da Costa ("aperfeiçoado" pelo  João Maldonado), da condição   n^2 + n  - 2.m^2 = 0  , onde  m natural.   Aplicando a dita fórmula de Bhaskara n = [-1 + sqrt(1+8.m^2)]

[obm-l] RE: [obm-l] Re: [obm-l] Questão de probabilidade(dúvida sobre gabarito)

2011-09-26 Por tôpico marcone augusto araújo borges
Mais uma idéia bem interessante. Date: Mon, 26 Sep 2011 14:25:40 -0300 Subject: [obm-l] Re: [obm-l] Questão de probabilidade(dúvida sobre gabarito) From: abrlw...@gmail.com To: obm-l@mat.puc-rio.br Ola' Marcone e colegas da lista, uma vez posicionada a 1a pessoa numa cadeira qualquer, as out

[obm-l] RE: [obm-l] Re: [obm-l] Questão de probabilidade(dúvida sobre gabarito)

2011-09-25 Por tôpico marcone augusto araújo borges
Ajudou e muito. Eu cometi um erro bobo ao não diferenciar as situações em que alguem começa sentando na primeira ou última cadeira e as outras situações. Antes mesmo de ler a sua enriquecedora(como sempre)mensagem,já tinha percebido o erro. De fato,considerando que poderiamos ter h(homem),m(mul

Re: [obm-l] Re: [obm-l] Re: [obm-l] Questão de geometria plana!! ajuda em n ova solução

2011-09-12 Por tôpico douglas . oliveira
nao conheco nao , se puder me dizer ..att douglas On Sat, 10 Sep 2011 16:25:19 -0500 (PET), Julio César Saldaña wrote: > você conhece a solução que usa congruência de triângulos e areas? > > Julio Saldaña > > -- Mensaje original --- > De : obm-l@mat.puc-rio.br [2] > Para : obm-l@m

[obm-l] Re: [obm-l] Re: [obm-l] Questão de geometria plana!! ajuda em nova solução

2011-09-12 Por tôpico Jefferson Franca
Que tal se vc unir este ponto aos vértices do triangulo, formando assim outros 3 triangulos menores tais que a soma das áreas deles seja a área do triangulo equilátero ? abs e boa sorte De: Johann Dirichlet Para: obm-l@mat.puc-rio.br Enviadas: Sábado, 10 de Se

[obm-l] RE: [obm-l] Re: [obm-l] Re: [obm-l] Questão de geometria plana!! ajuda em n ova solução

2011-09-10 Por tôpico João Maldonado
ana...@pucp.edu.pe > To: obm-l@mat.puc-rio.br > CC: > Subject: [obm-l] Re: [obm-l] Re: [obm-l] Questão de geometria plana!! ajuda > em n ova solução > Date: Sat, 10 Sep 2011 16:25:19 -0500 > > > > você conhece a solução que usa congruência de triângul

[obm-l] Re: [obm-l] Re: [obm-l] Questão de geometria plana!! ajuda em n ova solução

2011-09-10 Por tôpico Julio César Saldaña
você conhece a solução que usa congruência de triângulos e areas? Julio Saldaña -- Mensaje original --- De : obm-l@mat.puc-rio.br Para : obm-l@mat.puc-rio.br Fecha : Sat, 10 Sep 2011 17:17:42 -0300 Asunto : [obm-l] Re: [obm-l] Questão de geometria plana!! ajuda em n ova solução Ué, d

[obm-l] RE: [obm-l] Re: [obm-l] RE: [obm-l] Questão do colégio naval 2011

2011-09-01 Por tôpico João Maldonado
-0700 From: luizfelipec...@yahoo.com.br Subject: [obm-l] Re: [obm-l] RE: [obm-l] Questão do colégio naval 2011 To: obm-l@mat.puc-rio.br Ola João, O problema é que, pelo que lembro, na 8a. seria, não estudávmos congruencias modulares. Abs Felipe --- Em qua, 31/8/11, João Maldonado escreveu

[obm-l] Re: [obm-l] RE: [obm-l] Questão do colégio naval 2011

2011-08-31 Por tôpico luiz silva
Ola João,   O problema é que, pelo que lembro, na 8a. seria, não estudávmos congruencias modulares.   Abs Felipe --- Em qua, 31/8/11, João Maldonado escreveu: De: João Maldonado Assunto: [obm-l] RE: [obm-l] Questão do colégio naval 2011 Para: obm-l@mat.puc-rio.br Data: Quarta-feira, 31 de Ago

[obm-l] Re: [obm-l] Re: [obm-l] questão trigonometria complicada

2011-07-25 Por tôpico Jefferson Franca
Professor novamente agradeço ao senhor pela resolução VERDADEIRAMENTE inteligente e digna de um trabalhador!!! obrigado e abs De: Ralph Teixeira Para: obm-l@mat.puc-rio.br Enviadas: Segunda-feira, 27 de Junho de 2011 17:43 Assunto: [obm-l] Re: [obm-l] questão t

[obm-l] Re: [obm-l] Re: [obm-l] questão trigonometria complicada

2011-07-19 Por tôpico Jefferson Franca
Muito, muito , muito obrigado! Não foi fácil resolver. De: Ralph Teixeira Para: obm-l@mat.puc-rio.br Enviadas: Segunda-feira, 27 de Junho de 2011 17:52 Assunto: [obm-l] Re: [obm-l] questão trigonometria complicada Ah, já vi errinho de sinal no meio do caminho,

[obm-l] RES: [obm-l] Re: [obm-l] Re: [obm-l] questão trigonometria complicada

2011-06-28 Por tôpico André A.Seidel
Quero sair da lista obm-l -Mensagem original- De: owner-ob...@mat.puc-rio.br [mailto:owner-ob...@mat.puc-rio.br] Em nome de Johann Dirichlet Enviada em: terça-feira, 28 de junho de 2011 11:18 Para: obm-l@mat.puc-rio.br Assunto: [obm-l] Re: [obm-l] Re: [obm-l] questão trigonometria

[obm-l] Re: [obm-l] Re: [obm-l] questão trigonometria complicada

2011-06-28 Por tôpico Johann Dirichlet
Este foi um problema da revista Kvant, na verdade um artigo. Eis o site (pra quem encarar um russinho básico...) http://kvant.mccme.ru/ Em 27/06/11, Ralph Teixeira escreveu: > Ah, já vi errinho de sinal no meio do caminho, no sinal de ab+ac+bc e no de > abc. Corrigi abaixo, mas deve haver outros

[obm-l] Re: [obm-l] Re: [obm-l] RE: [obm-l] Questão de Química

2011-06-05 Por tôpico Eduardo Wilner
 f) estaria correta se não falasse em massa atômica; o número de massa é a soma dos números de prótons e neutrons.  --- Em dom, 5/6/11, Pierry �ngelo Pereira escreveu: De: Pierry �ngelo Pereira Assunto: [obm-l] Re: [obm-l] RE: [obm-l] Questão de Química Para: obm-l@mat.puc-rio.br Data

[obm-l] Re: [obm-l] RE: [obm-l] Questão de Química

2011-06-05 Por tôpico Pierry �ngelo Pereira
Obrigado João, ficou bem claro. Abraço. 2011/6/5 João Maldonado > Olá, > > > a) Verdadeira -> Prót ons e Nêutrons no núcleo, elétrons na eletrosfera > b) Verdadeira -> Bohr provou isso > c) Verdadeira -> Ânion é um íon negativo (mais elétrons de prótons), > Cátion é um íon positiv (mais pró

[obm-l] Re: [obm-l] Re: [obm-l] RE: [obm-l] Re: [obm-l] Questão - Teoria dos Nùmeros

2011-02-05 Por tôpico Willy George do Amaral Petrenko
Isso é bem mais simples. Não tinha visto a sua solução. 2011/2/5 Tiago > Suponha 2^n = 1 mod n. Se p é o *menor* primo que divide n, então 2^n = 1 > mod p. Pelo pequeno teorema de fermat, 2^(p-1) = 1 mod p. Se d=mdc(n,p-1), > então 2^d = 1 mod n. Mas p é o menor primo que divide n e d seja, 2 =

[obm-l] Re: [obm-l] RE: [obm-l] Re: [obm-l] Questão - Teoria dos Nùmeros

2011-02-05 Por tôpico Willy George do Amaral Petrenko
*Achar todos os naturais tais que (2^n-1)/n é inteiro.* Acho que vai ser complicado resolver isso com o t. fermat. Mas vasculhei as wikipedias da vida e encontrei o seguinte teorema, generalização do t. euler: http://en.wikipedia.org/wiki/Carmichael_function Basicamente ele aponta qual é o menor

[obm-l] Re: [obm-l] RE: [obm-l] Re: [obm-l] Questão - Teoria dos Nùmeros

2011-02-05 Por tôpico Tiago
Suponha 2^n = 1 mod n. Se p é o *menor* primo que divide n, então 2^n = 1 mod p. Pelo pequeno teorema de fermat, 2^(p-1) = 1 mod p. Se d=mdc(n,p-1), então 2^d = 1 mod n. Mas p é o menor primo que divide n e d > Oi Dinei, blz? Tow brincando com o cubo aki hehe! > > Se liga que a^(p-1) =1 (mod p) q

[obm-l] RE: [obm-l] RE: [obm-l] Re: [obm-l] Questão - Teoria dos Nùmeros

2011-02-05 Por tôpico Jordan Piva
trank dinei, zero stress... Agora tow esperando uma solução aí, cara tow maior tempao com essa questão e nada... Alguem ajuda aih pessoal: determinar todos os n naturarais, tal que (2^n-1)/n é inteiro. Subject: Re: [obm-l] RE: [obm-l] Re: [obm-l] Questão - Teoria dos Nùmeros From

[obm-l] RE: [obm-l] Re: [obm-l] Questão - Teoria dos Nùmeros

2011-02-05 Por tôpico Jordan Piva
Oi Dinei, blz? Tow brincando com o cubo aki hehe! Se liga que a^(p-1) =1 (mod p) qndo mdc(a,p)=1 blz, porque isso implica a^p=2 (mod p)? Tow mongolizando mto? Naum seria a^p=a (modp)? Date: Tue, 1 Feb 2011 17:28:45 -0200 Subject: [obm-l] Re: [obm-l] Questão - Teoria dos Nùmeros From: edward.el

Re: [obm-l] RE: [obm-l] Re: [obm-l] Questão - Teoria dos Nùmeros

2011-02-01 Por tôpico Diniz
Na verdade eu viajei Haha, misturei sem querer duas soluções q eu estava tentando, oq eu fiz esta absurdamente errado! Enviado via iPhone 4 Em 01/02/2011, às 18:46, João Maldonado escreveu: > > > > Date: Tue, 1 Feb 2011 17:28:45 -0200 > Subject: [obm-l] Re: [obm-l] Questão - Teoria dos Nù

[obm-l] RE: [obm-l] Re: [obm-l] Questão - Teoria dos Nùmeros

2011-02-01 Por tôpico João Maldonado
Date: Tue, 1 Feb 2011 17:28:45 -0200 Subject: [obm-l] Re: [obm-l] Questão - Teoria dos Nùmeros From: edward.elric...@gmail.com To: obm-l@mat.puc-rio.br Sabemos que n não pode ser par. Seja p um numero primo que divide n (n=p*n´). Temos que 2^n =1 (mod p), mas sabemos que a^(p-1)= 1 (mod

[obm-l] RE: [obm-l] Re: [obm -l] QUESTÃO AMT FINA NCEIRA

2010-12-14 Por tôpico Paulo Santa Rita
Olá a todos ! Total apoio ao que o Palmerim disse. Um abraço a todos PSR,31412100A35 Date: Mon, 13 Dec 2010 19:23:02 -0200 Subject: [obm-l] Re: [obm-l] QUESTÃO AMT FINANCEIRA From: palmerimsoa...@gmail.com To: obm-l@mat.puc-rio.br Oi Rogério, está parecendo que você tem uma lista de exercício

[obm-l] Re: [obm-l] Re: [obm-l] Re: [obm-l] RE: [obm-l] RE: [obm-l] questão básica de probabilidade

2010-10-25 Por tôpico Eduardo Farias
Obrigado a todos que responderam. Realmente explicações bem didáticas, abraço! Em 20 de outubro de 2010 22:24, Adalberto Dornelles escreveu: > Olá, > > Uma outra maneira de verificar que é 50% é perceber o seguinte "Espaço de > eventos" > > Numero de João > 1 2 3 4 5 ... 60 > N 1 . J

[obm-l] Re: [obm-l] RE: [obm-l] RE: [obm-l] questão básica de probabilidade

2010-10-20 Por tôpico Hugo Fernando Marques Fernandes
Se não há dois bilhetes iguais, então ocorre 1 de dois casos: o número de João é maior que o do Manuel ou vice-versa. Então: 1 caso favorável (João > Manoel) / 2 casos posíveis = 1/2 => 50%. Abraços Hugo. 2010/10/18 João Maldonado > De olho escolheria a letra D, mas se quisesse fazer contas

[obm-l] RE: [obm-l] RE: [obm -l] questão básica d e probabilidade

2010-10-18 Por tôpico João Maldonado
De olho escolheria a letra D, mas se quisesse fazer contas. Caso Manuel escolha o 1, João tem 59/59 chances de tirar um bilhete maior. Caso Manuel escolha o 2, João tem 58/59 chances de tirar um bilhete maior. . . . Caso Manuel escolha o 60, João tem 0/59 chances de tirar um bilhete maior. S

[obm-l] RE: [obm-l] RE: [obm -l] Questão do IME

2010-03-16 Por tôpico marcone augusto araújo borges
O primeiro membro é 5x^2+2y^2.Desculpe. From: leandrorec...@msn.com To: obm-l@mat.puc-rio.br Subject: [obm-l] RE: [obm-l] Questão do IME Date: Mon, 15 Mar 2010 20:45:21 -0700 Marcone, O enunciado esta correto? 5x^2+2x^2 no primeiro membro? Seria 7x^2? Podes confirmar? From: marconebo

Re: {Disarmed} [obm-l] Re: [obm-l] Re: [obm-l] Questão 8 da prova do ime

2009-11-04 Por tôpico Sergio Lima Netto
oi Felipe, Se q^2 < 144, entao p^n < 0. Acho que estas solucoes nao se aplicam, pois entendo que numero primo seja (de antemao) natural. Abraco, sergio On Tue, 3 Nov 2009 12:45:45 -0800 (PST), luiz silva wrote > Ola Sergio, >   > Eu deixei passar o caso em que q-12=1, e assim o caso em que p=5. 

[obm-l] Re: [obm-l] Re: [obm-l] Questão sobre poliinômios

2009-10-17 Por tôpico João Paulo V . Bonifácio
Obrigado pessoal! Abs! 2009/10/16 Ralph Teixeira > Temos: > P(x)=f(x)(x-2)(x-3)+R(x) onde R(x)=ax+b > e > P(x)=Q(x)(x-2)+h > > Fazendo x=2 em ambas, vem P(2)=2a+b=h, entao 2a+b=h=-1. > Fazendo x=3 em ambas, vem P(3)=3a+b=Q(3)+h, entao 3a+b=3-1=2. > Resolvendo o sistema em a e b, vem a=3 e b=-7.

[obm-l] Re: [obm-l] Re: [obm-l] questão de cônicas

2009-07-15 Por tôpico Carlos Alberto da Silva Victor
Ok, Nehab, De uma forma rigorosa deveria escolher t. AC.BD +k .AD.BC =0 para incluir todas . Obrigado e , como sempre, puxando a nossa orelha de uma forma bastante sutil. Abraços Carlos Victor 2009/7/15 Carlos Nehab > Oi, queridos amigos, > > (só para cutucar você, Alexandre) ... e per

  1   2   >